Proving that the eigenvalues of a Hermitian matrix is real

Click For Summary
SUMMARY

The eigenvalues of a Hermitian matrix are definitively real, as established by the properties of Hermitian matrices. The discussion references the relationship between a vector and its conjugate transpose, specifically noting that the product v*Av results in a Hermitian matrix. The assertion that v*v is Hermitian is supported by the definition of Hermitian matrices, which are equal to their conjugate transpose.

PREREQUISITES
  • Understanding of Hermitian matrices
  • Familiarity with eigenvalues and eigenvectors
  • Knowledge of conjugate transpose operations
  • Basic linear algebra concepts
NEXT STEPS
  • Study the properties of Hermitian matrices in linear algebra
  • Learn about the spectral theorem for Hermitian matrices
  • Explore proofs related to eigenvalues of Hermitian matrices
  • Investigate applications of Hermitian matrices in quantum mechanics
USEFUL FOR

Students and professionals in mathematics, particularly those studying linear algebra, as well as physicists and engineers working with quantum mechanics or systems involving Hermitian operators.

stgermaine
Messages
45
Reaction score
0

Homework Statement


Prove that the eigenvalues of a Hermitian matrix is real.
http://www.proofwiki.org/wiki/Hermitian_Matrix_has_Real_Eigenvalues

The website says that "By Product with Conjugate Transpose Matrix is Hermitian, v*v is Hermitian. " where v* is the conjugate transpose of v.

Homework Equations





The Attempt at a Solution



I'm not sure why that is true. v*Av is equal to v*v, and v*Av is a Hermitian matrix. Intuitively, v*v seems like a Hermitian matrix, but I need a real theorem that would show that.
 
Physics news on Phys.org
Do you really need a proof that v*v is Hermitian? Some things are just obvious. This is one of those things.
 
Question: A clock's minute hand has length 4 and its hour hand has length 3. What is the distance between the tips at the moment when it is increasing most rapidly?(Putnam Exam Question) Answer: Making assumption that both the hands moves at constant angular velocities, the answer is ## \sqrt{7} .## But don't you think this assumption is somewhat doubtful and wrong?

Similar threads

  • · Replies 2 ·
Replies
2
Views
3K
  • · Replies 4 ·
Replies
4
Views
3K
  • · Replies 13 ·
Replies
13
Views
11K
  • · Replies 31 ·
2
Replies
31
Views
4K
  • · Replies 3 ·
Replies
3
Views
2K
  • · Replies 13 ·
Replies
13
Views
3K
  • · Replies 4 ·
Replies
4
Views
4K
  • · Replies 1 ·
Replies
1
Views
1K
Replies
4
Views
2K
  • · Replies 3 ·
Replies
3
Views
1K